Calculate $lim_{nrightarrow infty }left(frac{n^{2}+1}{n+1}right)^{tfrac{n+1}{n^{2}+1}}$












2












$begingroup$



$$lim_{nrightarrow infty }left(frac{n^{2}+1}{n+1}right)^{tfrac{n+1}{n^{2}+1}}$$




I tried to use $f^{g}=e^{g ln f}$ and I got $e^{tfrac{n+1}{n^{2}+1}ln left(tfrac{n^2+1}{n+1} right)}$. How to continue ?










share|cite|improve this question











$endgroup$

















    2












    $begingroup$



    $$lim_{nrightarrow infty }left(frac{n^{2}+1}{n+1}right)^{tfrac{n+1}{n^{2}+1}}$$




    I tried to use $f^{g}=e^{g ln f}$ and I got $e^{tfrac{n+1}{n^{2}+1}ln left(tfrac{n^2+1}{n+1} right)}$. How to continue ?










    share|cite|improve this question











    $endgroup$















      2












      2








      2





      $begingroup$



      $$lim_{nrightarrow infty }left(frac{n^{2}+1}{n+1}right)^{tfrac{n+1}{n^{2}+1}}$$




      I tried to use $f^{g}=e^{g ln f}$ and I got $e^{tfrac{n+1}{n^{2}+1}ln left(tfrac{n^2+1}{n+1} right)}$. How to continue ?










      share|cite|improve this question











      $endgroup$





      $$lim_{nrightarrow infty }left(frac{n^{2}+1}{n+1}right)^{tfrac{n+1}{n^{2}+1}}$$




      I tried to use $f^{g}=e^{g ln f}$ and I got $e^{tfrac{n+1}{n^{2}+1}ln left(tfrac{n^2+1}{n+1} right)}$. How to continue ?







      calculus limits






      share|cite|improve this question















      share|cite|improve this question













      share|cite|improve this question




      share|cite|improve this question








      edited Jan 26 at 19:28









      TheSimpliFire

      12.5k62460




      12.5k62460










      asked Jan 26 at 18:46









      DaniVajaDaniVaja

      977




      977






















          4 Answers
          4






          active

          oldest

          votes


















          2












          $begingroup$

          Hint. Use substitution: set $x=dfrac{n^2+1}{n+1}$. What is the limit of $x$ when $n$ tends to $infty$?






          share|cite|improve this answer











          $endgroup$









          • 1




            $begingroup$
            I got $e^{lim_{xrightarrow 0}frac{lnx}{x}}=e^{0}=1$ is that right ?
            $endgroup$
            – DaniVaja
            Jan 26 at 19:00










          • $begingroup$
            Yes, absolutely.
            $endgroup$
            – Bernard
            Jan 26 at 19:02



















          4












          $begingroup$

          Write $m=frac{n^2+1}{n+1}$, so $mtoinfty$ as $ntoinfty$. Your limit is $$lim_{mtoinfty}expfrac{ln m}{m}=explim_{mtoinfty}frac{ln m}{m}=exp 0=1.$$






          share|cite|improve this answer









          $endgroup$













          • $begingroup$
            This is the same as what I propose, with more details.
            $endgroup$
            – Bernard
            Jan 26 at 19:01



















          0












          $begingroup$

          Making the problem more general, consider
          $$a_n=left(frac{n^{2}+a}{n+b}right)^{tfrac{n+c}{n^{2}+d}}implies log(a_n)={tfrac{n+c}{n^{2}+d}}logleft(frac{n^{2}+a}{n+b}right)$$ Now, use Taylor expansions for large $n$
          $$logleft(frac{n^{2}+a}{n+b}right)=log
          left({n}right)-frac{b}{n}+frac{a+frac{b^2}{2}}{n^2}+Oleft(frac{1}{n^3}right)$$

          $${tfrac{n+c}{n^{2}+d}}=frac{1}{n}+frac{c}{n^2}+Oleft(frac{1}{n^3}right)$$
          $$log(a_n)=frac{log left({n}right)}{n}+frac{-b+c log
          left({n}right)}{n^2}+Oleft(frac{1}{n^3}right)$$
          Continuing with Taylor
          $$a_n=e^{log(a_n)}=1+frac{log left({n}right)}{n}+frac{2 left(-b+c log
          left({n}right)right)+log ^2left({n}right)}{2
          n^2}+Oleft(frac{1}{n^3}right)$$

          Then $cdots$ $text{ ???}$ $forall {a,b,c,d}$






          share|cite|improve this answer









          $endgroup$





















            0












            $begingroup$

            $begin{array}{l}
            displaystylelim_{n to infty}left({n^{2} + 1 over n + 1}right)^{largeleft(n + 1right)/left(n^{2} + 1right)} =
            lim_{n to infty}n^{1/n} =
            expleft(lim_{n to infty}{lnleft(nright) over n}right) \[5mm] =
            displaystyle
            expleft(lim_{n to infty}
            {lnleft(n + 1right) - lnleft(n right) over left[n + 1right] - n}right) =
            expleft(lim_{n to infty}
            lnleft(1 + {1 over n}right)right) = expleft(0right) =
            bbox[10px,#ffd,border:1px groove navy]{1}
            end{array}
            $






            share|cite|improve this answer











            $endgroup$













              Your Answer





              StackExchange.ifUsing("editor", function () {
              return StackExchange.using("mathjaxEditing", function () {
              StackExchange.MarkdownEditor.creationCallbacks.add(function (editor, postfix) {
              StackExchange.mathjaxEditing.prepareWmdForMathJax(editor, postfix, [["$", "$"], ["\\(","\\)"]]);
              });
              });
              }, "mathjax-editing");

              StackExchange.ready(function() {
              var channelOptions = {
              tags: "".split(" "),
              id: "69"
              };
              initTagRenderer("".split(" "), "".split(" "), channelOptions);

              StackExchange.using("externalEditor", function() {
              // Have to fire editor after snippets, if snippets enabled
              if (StackExchange.settings.snippets.snippetsEnabled) {
              StackExchange.using("snippets", function() {
              createEditor();
              });
              }
              else {
              createEditor();
              }
              });

              function createEditor() {
              StackExchange.prepareEditor({
              heartbeatType: 'answer',
              autoActivateHeartbeat: false,
              convertImagesToLinks: true,
              noModals: true,
              showLowRepImageUploadWarning: true,
              reputationToPostImages: 10,
              bindNavPrevention: true,
              postfix: "",
              imageUploader: {
              brandingHtml: "Powered by u003ca class="icon-imgur-white" href="https://imgur.com/"u003eu003c/au003e",
              contentPolicyHtml: "User contributions licensed under u003ca href="https://creativecommons.org/licenses/by-sa/3.0/"u003ecc by-sa 3.0 with attribution requiredu003c/au003e u003ca href="https://stackoverflow.com/legal/content-policy"u003e(content policy)u003c/au003e",
              allowUrls: true
              },
              noCode: true, onDemand: true,
              discardSelector: ".discard-answer"
              ,immediatelyShowMarkdownHelp:true
              });


              }
              });














              draft saved

              draft discarded


















              StackExchange.ready(
              function () {
              StackExchange.openid.initPostLogin('.new-post-login', 'https%3a%2f%2fmath.stackexchange.com%2fquestions%2f3088597%2fcalculate-lim-n-rightarrow-infty-left-fracn21n1-right-tfracn%23new-answer', 'question_page');
              }
              );

              Post as a guest















              Required, but never shown

























              4 Answers
              4






              active

              oldest

              votes








              4 Answers
              4






              active

              oldest

              votes









              active

              oldest

              votes






              active

              oldest

              votes









              2












              $begingroup$

              Hint. Use substitution: set $x=dfrac{n^2+1}{n+1}$. What is the limit of $x$ when $n$ tends to $infty$?






              share|cite|improve this answer











              $endgroup$









              • 1




                $begingroup$
                I got $e^{lim_{xrightarrow 0}frac{lnx}{x}}=e^{0}=1$ is that right ?
                $endgroup$
                – DaniVaja
                Jan 26 at 19:00










              • $begingroup$
                Yes, absolutely.
                $endgroup$
                – Bernard
                Jan 26 at 19:02
















              2












              $begingroup$

              Hint. Use substitution: set $x=dfrac{n^2+1}{n+1}$. What is the limit of $x$ when $n$ tends to $infty$?






              share|cite|improve this answer











              $endgroup$









              • 1




                $begingroup$
                I got $e^{lim_{xrightarrow 0}frac{lnx}{x}}=e^{0}=1$ is that right ?
                $endgroup$
                – DaniVaja
                Jan 26 at 19:00










              • $begingroup$
                Yes, absolutely.
                $endgroup$
                – Bernard
                Jan 26 at 19:02














              2












              2








              2





              $begingroup$

              Hint. Use substitution: set $x=dfrac{n^2+1}{n+1}$. What is the limit of $x$ when $n$ tends to $infty$?






              share|cite|improve this answer











              $endgroup$



              Hint. Use substitution: set $x=dfrac{n^2+1}{n+1}$. What is the limit of $x$ when $n$ tends to $infty$?







              share|cite|improve this answer














              share|cite|improve this answer



              share|cite|improve this answer








              edited Jan 26 at 18:57

























              answered Jan 26 at 18:53









              BernardBernard

              121k740116




              121k740116








              • 1




                $begingroup$
                I got $e^{lim_{xrightarrow 0}frac{lnx}{x}}=e^{0}=1$ is that right ?
                $endgroup$
                – DaniVaja
                Jan 26 at 19:00










              • $begingroup$
                Yes, absolutely.
                $endgroup$
                – Bernard
                Jan 26 at 19:02














              • 1




                $begingroup$
                I got $e^{lim_{xrightarrow 0}frac{lnx}{x}}=e^{0}=1$ is that right ?
                $endgroup$
                – DaniVaja
                Jan 26 at 19:00










              • $begingroup$
                Yes, absolutely.
                $endgroup$
                – Bernard
                Jan 26 at 19:02








              1




              1




              $begingroup$
              I got $e^{lim_{xrightarrow 0}frac{lnx}{x}}=e^{0}=1$ is that right ?
              $endgroup$
              – DaniVaja
              Jan 26 at 19:00




              $begingroup$
              I got $e^{lim_{xrightarrow 0}frac{lnx}{x}}=e^{0}=1$ is that right ?
              $endgroup$
              – DaniVaja
              Jan 26 at 19:00












              $begingroup$
              Yes, absolutely.
              $endgroup$
              – Bernard
              Jan 26 at 19:02




              $begingroup$
              Yes, absolutely.
              $endgroup$
              – Bernard
              Jan 26 at 19:02











              4












              $begingroup$

              Write $m=frac{n^2+1}{n+1}$, so $mtoinfty$ as $ntoinfty$. Your limit is $$lim_{mtoinfty}expfrac{ln m}{m}=explim_{mtoinfty}frac{ln m}{m}=exp 0=1.$$






              share|cite|improve this answer









              $endgroup$













              • $begingroup$
                This is the same as what I propose, with more details.
                $endgroup$
                – Bernard
                Jan 26 at 19:01
















              4












              $begingroup$

              Write $m=frac{n^2+1}{n+1}$, so $mtoinfty$ as $ntoinfty$. Your limit is $$lim_{mtoinfty}expfrac{ln m}{m}=explim_{mtoinfty}frac{ln m}{m}=exp 0=1.$$






              share|cite|improve this answer









              $endgroup$













              • $begingroup$
                This is the same as what I propose, with more details.
                $endgroup$
                – Bernard
                Jan 26 at 19:01














              4












              4








              4





              $begingroup$

              Write $m=frac{n^2+1}{n+1}$, so $mtoinfty$ as $ntoinfty$. Your limit is $$lim_{mtoinfty}expfrac{ln m}{m}=explim_{mtoinfty}frac{ln m}{m}=exp 0=1.$$






              share|cite|improve this answer









              $endgroup$



              Write $m=frac{n^2+1}{n+1}$, so $mtoinfty$ as $ntoinfty$. Your limit is $$lim_{mtoinfty}expfrac{ln m}{m}=explim_{mtoinfty}frac{ln m}{m}=exp 0=1.$$







              share|cite|improve this answer












              share|cite|improve this answer



              share|cite|improve this answer










              answered Jan 26 at 19:00









              J.G.J.G.

              27.1k22843




              27.1k22843












              • $begingroup$
                This is the same as what I propose, with more details.
                $endgroup$
                – Bernard
                Jan 26 at 19:01


















              • $begingroup$
                This is the same as what I propose, with more details.
                $endgroup$
                – Bernard
                Jan 26 at 19:01
















              $begingroup$
              This is the same as what I propose, with more details.
              $endgroup$
              – Bernard
              Jan 26 at 19:01




              $begingroup$
              This is the same as what I propose, with more details.
              $endgroup$
              – Bernard
              Jan 26 at 19:01











              0












              $begingroup$

              Making the problem more general, consider
              $$a_n=left(frac{n^{2}+a}{n+b}right)^{tfrac{n+c}{n^{2}+d}}implies log(a_n)={tfrac{n+c}{n^{2}+d}}logleft(frac{n^{2}+a}{n+b}right)$$ Now, use Taylor expansions for large $n$
              $$logleft(frac{n^{2}+a}{n+b}right)=log
              left({n}right)-frac{b}{n}+frac{a+frac{b^2}{2}}{n^2}+Oleft(frac{1}{n^3}right)$$

              $${tfrac{n+c}{n^{2}+d}}=frac{1}{n}+frac{c}{n^2}+Oleft(frac{1}{n^3}right)$$
              $$log(a_n)=frac{log left({n}right)}{n}+frac{-b+c log
              left({n}right)}{n^2}+Oleft(frac{1}{n^3}right)$$
              Continuing with Taylor
              $$a_n=e^{log(a_n)}=1+frac{log left({n}right)}{n}+frac{2 left(-b+c log
              left({n}right)right)+log ^2left({n}right)}{2
              n^2}+Oleft(frac{1}{n^3}right)$$

              Then $cdots$ $text{ ???}$ $forall {a,b,c,d}$






              share|cite|improve this answer









              $endgroup$


















                0












                $begingroup$

                Making the problem more general, consider
                $$a_n=left(frac{n^{2}+a}{n+b}right)^{tfrac{n+c}{n^{2}+d}}implies log(a_n)={tfrac{n+c}{n^{2}+d}}logleft(frac{n^{2}+a}{n+b}right)$$ Now, use Taylor expansions for large $n$
                $$logleft(frac{n^{2}+a}{n+b}right)=log
                left({n}right)-frac{b}{n}+frac{a+frac{b^2}{2}}{n^2}+Oleft(frac{1}{n^3}right)$$

                $${tfrac{n+c}{n^{2}+d}}=frac{1}{n}+frac{c}{n^2}+Oleft(frac{1}{n^3}right)$$
                $$log(a_n)=frac{log left({n}right)}{n}+frac{-b+c log
                left({n}right)}{n^2}+Oleft(frac{1}{n^3}right)$$
                Continuing with Taylor
                $$a_n=e^{log(a_n)}=1+frac{log left({n}right)}{n}+frac{2 left(-b+c log
                left({n}right)right)+log ^2left({n}right)}{2
                n^2}+Oleft(frac{1}{n^3}right)$$

                Then $cdots$ $text{ ???}$ $forall {a,b,c,d}$






                share|cite|improve this answer









                $endgroup$
















                  0












                  0








                  0





                  $begingroup$

                  Making the problem more general, consider
                  $$a_n=left(frac{n^{2}+a}{n+b}right)^{tfrac{n+c}{n^{2}+d}}implies log(a_n)={tfrac{n+c}{n^{2}+d}}logleft(frac{n^{2}+a}{n+b}right)$$ Now, use Taylor expansions for large $n$
                  $$logleft(frac{n^{2}+a}{n+b}right)=log
                  left({n}right)-frac{b}{n}+frac{a+frac{b^2}{2}}{n^2}+Oleft(frac{1}{n^3}right)$$

                  $${tfrac{n+c}{n^{2}+d}}=frac{1}{n}+frac{c}{n^2}+Oleft(frac{1}{n^3}right)$$
                  $$log(a_n)=frac{log left({n}right)}{n}+frac{-b+c log
                  left({n}right)}{n^2}+Oleft(frac{1}{n^3}right)$$
                  Continuing with Taylor
                  $$a_n=e^{log(a_n)}=1+frac{log left({n}right)}{n}+frac{2 left(-b+c log
                  left({n}right)right)+log ^2left({n}right)}{2
                  n^2}+Oleft(frac{1}{n^3}right)$$

                  Then $cdots$ $text{ ???}$ $forall {a,b,c,d}$






                  share|cite|improve this answer









                  $endgroup$



                  Making the problem more general, consider
                  $$a_n=left(frac{n^{2}+a}{n+b}right)^{tfrac{n+c}{n^{2}+d}}implies log(a_n)={tfrac{n+c}{n^{2}+d}}logleft(frac{n^{2}+a}{n+b}right)$$ Now, use Taylor expansions for large $n$
                  $$logleft(frac{n^{2}+a}{n+b}right)=log
                  left({n}right)-frac{b}{n}+frac{a+frac{b^2}{2}}{n^2}+Oleft(frac{1}{n^3}right)$$

                  $${tfrac{n+c}{n^{2}+d}}=frac{1}{n}+frac{c}{n^2}+Oleft(frac{1}{n^3}right)$$
                  $$log(a_n)=frac{log left({n}right)}{n}+frac{-b+c log
                  left({n}right)}{n^2}+Oleft(frac{1}{n^3}right)$$
                  Continuing with Taylor
                  $$a_n=e^{log(a_n)}=1+frac{log left({n}right)}{n}+frac{2 left(-b+c log
                  left({n}right)right)+log ^2left({n}right)}{2
                  n^2}+Oleft(frac{1}{n^3}right)$$

                  Then $cdots$ $text{ ???}$ $forall {a,b,c,d}$







                  share|cite|improve this answer












                  share|cite|improve this answer



                  share|cite|improve this answer










                  answered Jan 27 at 3:06









                  Claude LeiboviciClaude Leibovici

                  121k1157134




                  121k1157134























                      0












                      $begingroup$

                      $begin{array}{l}
                      displaystylelim_{n to infty}left({n^{2} + 1 over n + 1}right)^{largeleft(n + 1right)/left(n^{2} + 1right)} =
                      lim_{n to infty}n^{1/n} =
                      expleft(lim_{n to infty}{lnleft(nright) over n}right) \[5mm] =
                      displaystyle
                      expleft(lim_{n to infty}
                      {lnleft(n + 1right) - lnleft(n right) over left[n + 1right] - n}right) =
                      expleft(lim_{n to infty}
                      lnleft(1 + {1 over n}right)right) = expleft(0right) =
                      bbox[10px,#ffd,border:1px groove navy]{1}
                      end{array}
                      $






                      share|cite|improve this answer











                      $endgroup$


















                        0












                        $begingroup$

                        $begin{array}{l}
                        displaystylelim_{n to infty}left({n^{2} + 1 over n + 1}right)^{largeleft(n + 1right)/left(n^{2} + 1right)} =
                        lim_{n to infty}n^{1/n} =
                        expleft(lim_{n to infty}{lnleft(nright) over n}right) \[5mm] =
                        displaystyle
                        expleft(lim_{n to infty}
                        {lnleft(n + 1right) - lnleft(n right) over left[n + 1right] - n}right) =
                        expleft(lim_{n to infty}
                        lnleft(1 + {1 over n}right)right) = expleft(0right) =
                        bbox[10px,#ffd,border:1px groove navy]{1}
                        end{array}
                        $






                        share|cite|improve this answer











                        $endgroup$
















                          0












                          0








                          0





                          $begingroup$

                          $begin{array}{l}
                          displaystylelim_{n to infty}left({n^{2} + 1 over n + 1}right)^{largeleft(n + 1right)/left(n^{2} + 1right)} =
                          lim_{n to infty}n^{1/n} =
                          expleft(lim_{n to infty}{lnleft(nright) over n}right) \[5mm] =
                          displaystyle
                          expleft(lim_{n to infty}
                          {lnleft(n + 1right) - lnleft(n right) over left[n + 1right] - n}right) =
                          expleft(lim_{n to infty}
                          lnleft(1 + {1 over n}right)right) = expleft(0right) =
                          bbox[10px,#ffd,border:1px groove navy]{1}
                          end{array}
                          $






                          share|cite|improve this answer











                          $endgroup$



                          $begin{array}{l}
                          displaystylelim_{n to infty}left({n^{2} + 1 over n + 1}right)^{largeleft(n + 1right)/left(n^{2} + 1right)} =
                          lim_{n to infty}n^{1/n} =
                          expleft(lim_{n to infty}{lnleft(nright) over n}right) \[5mm] =
                          displaystyle
                          expleft(lim_{n to infty}
                          {lnleft(n + 1right) - lnleft(n right) over left[n + 1right] - n}right) =
                          expleft(lim_{n to infty}
                          lnleft(1 + {1 over n}right)right) = expleft(0right) =
                          bbox[10px,#ffd,border:1px groove navy]{1}
                          end{array}
                          $







                          share|cite|improve this answer














                          share|cite|improve this answer



                          share|cite|improve this answer








                          edited Jan 28 at 6:42

























                          answered Jan 28 at 2:27









                          Felix MarinFelix Marin

                          68k7108142




                          68k7108142






























                              draft saved

                              draft discarded




















































                              Thanks for contributing an answer to Mathematics Stack Exchange!


                              • Please be sure to answer the question. Provide details and share your research!

                              But avoid



                              • Asking for help, clarification, or responding to other answers.

                              • Making statements based on opinion; back them up with references or personal experience.


                              Use MathJax to format equations. MathJax reference.


                              To learn more, see our tips on writing great answers.




                              draft saved


                              draft discarded














                              StackExchange.ready(
                              function () {
                              StackExchange.openid.initPostLogin('.new-post-login', 'https%3a%2f%2fmath.stackexchange.com%2fquestions%2f3088597%2fcalculate-lim-n-rightarrow-infty-left-fracn21n1-right-tfracn%23new-answer', 'question_page');
                              }
                              );

                              Post as a guest















                              Required, but never shown





















































                              Required, but never shown














                              Required, but never shown












                              Required, but never shown







                              Required, but never shown

































                              Required, but never shown














                              Required, but never shown












                              Required, but never shown







                              Required, but never shown







                              Popular posts from this blog

                              flock() on closed filehandle LOCK_FILE at /usr/bin/apt-mirror

                              Mangá

                               ⁒  ․,‪⁊‑⁙ ⁖, ⁇‒※‌, †,⁖‗‌⁝    ‾‸⁘,‖⁔⁣,⁂‾
”‑,‥–,‬ ,⁀‹⁋‴⁑ ‒ ,‴⁋”‼ ⁨,‷⁔„ ‰′,‐‚ ‥‡‎“‷⁃⁨⁅⁣,⁔
⁇‘⁔⁡⁏⁌⁡‿‶‏⁨ ⁣⁕⁖⁨⁩⁥‽⁀  ‴‬⁜‟ ⁃‣‧⁕‮ …‍⁨‴ ⁩,⁚⁖‫ ,‵ ⁀,‮⁝‣‣ ⁑  ⁂– ․, ‾‽ ‏⁁“⁗‸ ‾… ‹‡⁌⁎‸‘ ‡⁏⁌‪ ‵⁛ ‎⁨ ―⁦⁤⁄⁕